0
$\begingroup$

How does the MacMahon function for counting plane partitions

$M(q) = \frac{1}{(1-q^n)^n}$

behave under modular transformations? For instance for $q= e^{2 \pi i \tau}$ where $\tau \rightarrow -1/\tau$.

$\endgroup$
3
  • $\begingroup$ conformal-field-theory? string-theory?? $\endgroup$ Commented Jun 25, 2024 at 1:28
  • $\begingroup$ According to arxiv.org/pdf/2109.15145 it is not related to a weakly modular form. Is that what you wanted to know? $\endgroup$ Commented Jun 25, 2024 at 3:59
  • 1
    $\begingroup$ Do you want a product sign somewhere? $\endgroup$ Commented Jun 25, 2024 at 17:29

0

You must log in to answer this question.

Start asking to get answers

Find the answer to your question by asking.

Ask question

Explore related questions

See similar questions with these tags.